Peskin & Schroeder Problem 4.1

In summary, Einj's hints were in the right direction, but to see it clearly we might need to go back a couple of steps and write out ##P(0)## in terms of contracted ##\phi(x)## and ##\phi(y)##. The next step is to replace the contraction with the Feynman propagator, which (I guess) you've tried to do in your ##A_3## expression. However, you didn't say how your ##A_3## relates to the desired ##P(0)## in the original question. A review of the material on p31 of P+S might also be helpful.
  • #1
ChrisVer
Gold Member
3,378
464
I am having some problems in evaluating the current problem's question (b)...

I have reached the point (writing only the term which I have problem with):

[itex]A_{3}= - \frac{1}{2} \int_{t_{0}}^{t} dt' dt'' \int d^{3}x \int d^{3}y j(x,t') j(y,t'') D_{F}(x-y)[/itex]

So for some unknown reasons, I cannot see that I can find the Fourier Transform of the classical source to bring it in the needed form...
eg.
[itex] D_{F}= \int \frac{d^{4}p}{(2 \pi)^{4}} \frac{i}{p^{2}-m^{2}} e^{i p (x-y)}[/itex]

[itex]A_{3}=- \frac{1}{2} \int d^{4}x \int d^{4}y j(x) j(y) \int \frac{d^{4}p}{(2 \pi)^{4}} \frac{1}{p^{2}-m^{2}} e^{i p x} e^{-ipy}[/itex][itex]A_{3}=- \frac{1}{2} \int \frac{d^{4}p}{(2 \pi)^{4}} \frac{1}{p^{2}-m^{2}} \int d^{4}x j(x) e^{i p x}\int d^{4}y j(y) e^{-ipy}[/itex]

However I don't know how this [itex]p^{0}=E_{p}[/itex] integration can be performed...
 
Last edited:
Physics news on Phys.org
  • #2
I'm not sure I understand what your question is. However the p0 integration is usually performed as follows:
$$
\int \frac{d^4p}{(2\pi)^4}\frac{e^{ip(x-y)}}{p_0^2-(\vec p^2+m^2)}=\int \frac{d^4p}{(2\pi)^4}\frac{e^{ip_0(x-y)_0}}{(p_0-E_p)(p_0+E_p)}e^{-i\vec p\cdot(\vec x-\vec y)},
$$
where [itex]E_p=\vec p^2+m^2[/itex]. In order for the integral to converge you need to close in the upper-half plane, i.e. you get the residue at [itex]p_0=E_p[/itex] and hence the final result is:
$$
i\int \frac{d^3p}{(2\pi)^32E_p}e^{ip(x-y)},
$$
where now [itex]p_0=E_p=\vec p^2+m^2[/itex].

Is this what you were looking for?
 
  • #3
yes but unfortunately i also have the [itex]j(p)=j(p^{0},\vec{p})[/itex] in the integrand...
 
  • #4
Why do you say so? From what I see your sources depend on x and y not on p.
 
  • #5
[itex] \int d^{4}x j(x) e^{i p x}= j(p)[/itex]
it;s the Fourier tranf of j
 
  • #6
Yes, exactly. But once you write that all the p dependence is in the exponential [itex]e^{ip(x-y)}[/itex] which I considered in my solution.
 
  • #7
ChrisVer,

I slogged through these problems ages ago, so I've pulled out my old workbook to refresh my memory. I approached it similarly to you, but I now realize it was subtly wrong. :redface:

At least I now know the fully correct solution. It's easy to fudge a correct-looking solution, but a truly correct solution is rather more difficult. :frown:
There are some tricky bits involving Feynman contours and real parts, etc, which are easy to overlook (as I discovered the hard way).

I'm not sure whether you want step-by-step help like a homework question, or just the final solution. I'll assume the former unless you say otherwise...

Einj's hints were in the right direction, but to see it clearly we might need to go back a couple of steps and write out ##P(0)## in terms of contracted ##\phi(x)## and ##\phi(y)##. The next step is to replace the contraction with the Feynman propagator, which (I guess) you've tried to do in your ##A_3## expression. But you didn't say how your ##A_3## relates to the desired ##P(0)## in the original question. (Getting that right makes an important difference.) A review of the material on p31 of P+S might also be helpful. :wink:

HTH.
 
Last edited:
  • #8
strangerep said:
But at least I now know the fully correct solution. It's easy to fudge a correct-looking solution, but a truly correct solution is rather more difficult. :frown:
There are some tricky bits involving Feynman contours and real parts, etc, which are easy to overlook (as I discovered the hard way).

Yes, you are right. As you can see I took the pole on the upper-half plane even though the zeros of my denominator both lies on the real axis. I haven't explicitly said that I was taking the right Feynman path, I am sorry I didn't mention that.
 
  • #9
my problem with Einj is that:
if you firstly do the integration of the Feynman propagator, without caring for the sources [itex]j(x)[/itex], then you will lose/have integrated the [itex]p^{0}[/itex] from the exponential and so you are left with the exponential with the 3-momentum on it. On the other hand, my sources also depend on the [itex]x^{0}=t[/itex] variable, and missing that I can't bring them to the Fourier transformed expression...
If I try to write them as:
[itex] \int dt j(\vec{x},t) [/itex]
then I'll probably get (I am not sure) after FT:
[itex] \int dt j(\vec{p},t)[/itex]
which is weird- I don't want the integral over time...

But I think I found a way out- Probably I can do the integral:
[itex]\int d^{4}p |j(p)|^{2} [propagator] [/itex]
The only problem with that is I don't know what will happen to [itex]j[/itex] when I look for the Residue.

(of course I did that strangerep- I just gave the final expression I'm trying to compute... I am not yet able to see through what kind of contractions happen without first writing the Wick's theorem on them)
 
  • #10
ChrisVer said:
my problem with Einj is that:
if you firstly do the integration of the Feynman propagator, without caring for the sources [itex]j(x)[/itex], then you will lose/have integrated the [itex]p^{0}[/itex] from the exponential and so you are left with the exponential with the 3-momentum on it. On the other hand, my sources also depend on the [itex]x^{0}=t[/itex] variable, and missing that I can't bring them to the Fourier transformed expression...
But did you actually attempt what Einj suggested, or are you just guessing about what might go wrong?

If you do the integration of the Feynman propagator carefully, you should still have factor(s) like $$e^{\pm i E_p (x^0 - y^0)}$$which should help with that perceived problem.

If I try to write them as:
[itex] \int dt j(\vec{x},t) [/itex]
then I'll probably get (I am not sure) after FT:
[itex] \int dt j(\vec{p},t)[/itex]
which is weird- I don't want the integral over time...

But I think I found a way out- Probably I can do the integral:
[itex]\int d^{4}p |j(p)|^{2} [propagator] [/itex]
The only problem with that is I don't know what will happen to [itex]j[/itex] when I look for the Residue.
It's possible to do it this way (that was my first method). One must make certain assumptions about convergence of ##j## anyway (i.e., that it vanishes fast enough on the large semicircle part of the contour). So one can perform that ##p^0## contour integral by standard methods.

Personally, I think it's easier to do the ##D_F## integration first -- provided you do it carefully.

(of course I did that strangerep- [...]
Hmm. Sounds like I'm not the right person to help you. (I'm not a mind reader.)
 
Last edited:
  • #11
strangerep said:
Hmm. Sounds like I'm not the right person to help you. (I'm not a mind reader.)

Sorry I didn't mean it like that... I just explained that there was no other way for me to write [itex]A_{3}[/itex] as I gave it, but by having followed the way you indicated before doing the contraction of the field etc :) nothing more/less...

"but to see it clearly we might need to go back a couple of steps and write out P(0) in terms of contracted ϕ(x) and ϕ(y). The next step is to replace the contraction with the Feynman propagator, which (I guess) you've tried to do in your A3 expression. But you didn't say how your A3 relates to the desired P(0) in the original question. (Getting that right makes an important difference.)"

Sorry if it didn't sound "right"/ sounded impolite ...

So, to make it more clear, the [itex]A_{3}[/itex] is just a part of the expansion of the amplitude...
[itex]A= 1+ A_{3} [/itex]
the 1 middle term which appears, dies because the field has zero vev.
Then the probability [itex]P(0)[/itex] will be just the module square of the amplitude:
[itex]P(0)= |1+A_{3}|^{2}[/itex]
 
  • #12
ChrisVer said:
Sorry I didn't mean it like that...
OK. Let us know if you need further clues.
 

1. What is Peskin & Schroeder Problem 4.1?

Peskin & Schroeder Problem 4.1 is a physics problem from the textbook "An Introduction to Quantum Field Theory" by Michael E. Peskin and Daniel V. Schroeder. It involves calculating the Feynman rules for scalar fields in quantum field theory.

2. Why is Peskin & Schroeder Problem 4.1 important?

This problem is important because it introduces students to the basics of quantum field theory, which is a fundamental theory in modern physics. It also helps students understand how to calculate particle interactions and scattering amplitudes.

3. What are the main concepts covered in Peskin & Schroeder Problem 4.1?

The main concepts covered in this problem include scalar fields, Feynman rules, and the propagator function. Students will also learn about complex numbers, integrals, and how to use Feynman diagrams to visualize particle interactions.

4. How difficult is Peskin & Schroeder Problem 4.1?

The difficulty of this problem can vary depending on the student's background and level of understanding of quantum field theory. However, it is generally considered a moderately challenging problem that requires a good understanding of the concepts and mathematical techniques used in quantum field theory.

5. Are there any helpful resources for solving Peskin & Schroeder Problem 4.1?

There are several helpful resources available for solving this problem, including online forums and study groups, textbooks on quantum field theory, and online tutorials and videos. It is also recommended to seek guidance from a professor or tutor if needed.

Similar threads

  • High Energy, Nuclear, Particle Physics
Replies
3
Views
2K
  • High Energy, Nuclear, Particle Physics
Replies
8
Views
2K
  • High Energy, Nuclear, Particle Physics
Replies
3
Views
2K
  • High Energy, Nuclear, Particle Physics
Replies
3
Views
867
  • High Energy, Nuclear, Particle Physics
Replies
3
Views
2K
  • High Energy, Nuclear, Particle Physics
Replies
3
Views
1K
  • High Energy, Nuclear, Particle Physics
Replies
5
Views
1K
  • High Energy, Nuclear, Particle Physics
Replies
2
Views
2K
  • High Energy, Nuclear, Particle Physics
Replies
6
Views
2K
  • High Energy, Nuclear, Particle Physics
Replies
5
Views
2K
Back
Top